Ditrama population

This topic has expert replies
Master | Next Rank: 500 Posts
Posts: 182
Joined: Sat Apr 11, 2009 4:55 am
Thanked: 11 times

Ditrama population

by Giorgio » Thu Jan 14, 2010 8:48 am
Ditrama is a federation made up of three autonomous regions: Korva, Mitro, and Guadar. Under the federal revenue-sharing plan, each region receives a share of federal revenues equal to the share of the total population of Ditrama residing in that region, as shown by a yearly population survey. Last year, the percentage of federal revenues Korva received for its share decreased somewhat even though the population survey on which the revenue-sharing was based showed that Korva's population had increased. If the statements above are true, which one of the following must also have been shown by the
population survey on which last year's revenue-sharing in Ditrama was based?

(A) Of the three regions, Korva had the smallest number of residents.

(B) The population of Korva grew by a smaller percentage than it did in previous years.

(C) The populations of Mitro and Guadar each increased by a percentage that exceeded the percentage by which the population of Korva increased.

(D) Of the three regions, Korva's numerical increase in population was the smallest.

(E) Korva's population grew by a smaller percentage than did the population of at least one of the other two autonomous
regions.

I will provide OA bit later!

GMAT Instructor
Posts: 1302
Joined: Mon Oct 19, 2009 2:13 pm
Location: Toronto
Thanked: 539 times
Followed by:164 members
GMAT Score:800

by Testluv » Thu Jan 14, 2010 10:25 am
Hi,

This is an inference question. The right answer to an inference question is something that must be true based on one or more statements in the passage. Here, we can make a deduction, and predict the correct answer.

We know that Korva's revenue share decreased even though their population increased. This must mean that, collectively, the other two countries experienced a greater proportional increase in population. Now, a key thing to understand is that in order for the other two countries to have experienced a collective increase, it doesn't have to be the case that both of these countries did. But at least one of them would have had to. Thus, choice E is correct.

_________

In general, in inference questions you should avoid choices that use extreme language, and be partial to choices whose word usage is more tentative; notice that choice E says "at least one", which makes it less categorical, more tentative.
Kaplan Teacher in Toronto

Master | Next Rank: 500 Posts
Posts: 182
Joined: Sat Apr 11, 2009 4:55 am
Thanked: 11 times

by Giorgio » Thu Jan 14, 2010 10:46 am
Testluv wrote:Hi,

This is an inference question. The right answer to an inference question is something that must be true based on one or more statements in the passage. Here, we can make a deduction, and predict the correct answer.

We know that Korva's revenue share decreased even though their population increased. This must mean that, collectively, the other two countries experienced a greater proportional increase in population. Now, a key thing to understand is that in order for the other two countries to have experienced a collective increase, it doesn't have to be the case that both of these countries did. But at least one of them would have had to. Thus, choice E is correct.

_________

In general, in inference questions you should avoid choices that use extreme language, and be partial to choices whose word usage is more tentative; notice that choice E says "at least one", which makes it less categorical, more tentative.
OA is E

Thanks Testluv for your explanation , however i don't understand the bolded part of your explanation, is there any chance to clarify this issue in numerical terms? This CR nearly killed me !

GMAT Instructor
Posts: 1302
Joined: Mon Oct 19, 2009 2:13 pm
Location: Toronto
Thanked: 539 times
Followed by:164 members
GMAT Score:800

by Testluv » Thu Jan 14, 2010 12:48 pm
sure.


................Before........After
Korvo........30...............40

Mitro.........30................30

Guadar.....40...............130

Total.........100.............200

As you can see even though Korvo's population increased, it's share of the total went down even though Guadar is the only other country that has experienced an increase.
Kaplan Teacher in Toronto

Master | Next Rank: 500 Posts
Posts: 182
Joined: Sat Apr 11, 2009 4:55 am
Thanked: 11 times

by Giorgio » Thu Jan 14, 2010 10:08 pm
But is not it possible that both of the populations increased?

GMAT Instructor
Posts: 1302
Joined: Mon Oct 19, 2009 2:13 pm
Location: Toronto
Thanked: 539 times
Followed by:164 members
GMAT Score:800

by Testluv » Thu Jan 14, 2010 10:11 pm
Giorgio wrote:But is not it possible that both of the populations increased?
Yep, sure is....but the question is asking for something that MUST be true. (in fact, all four of the wrong answers are "possible").
Kaplan Teacher in Toronto

Legendary Member
Posts: 503
Joined: Sun Aug 09, 2009 9:53 pm
Thanked: 31 times
Followed by:2 members

by mmslf75 » Fri Jan 15, 2010 12:59 am
Giorgio wrote:But is not it possible that both of the populations increased?
if I am not wrong, this is from PowerScore, number percentages chapter ??!

Master | Next Rank: 500 Posts
Posts: 182
Joined: Sat Apr 11, 2009 4:55 am
Thanked: 11 times

by Giorgio » Fri Jan 15, 2010 1:07 am
Exactly!

GMAT Instructor
Posts: 1302
Joined: Mon Oct 19, 2009 2:13 pm
Location: Toronto
Thanked: 539 times
Followed by:164 members
GMAT Score:800

by Testluv » Fri Jan 15, 2010 1:16 am
This is actually an LSAT question that I've come across more than a few times during the course of tutoring students for the LSAT.

But, as far as LSAT questions go, this is actually a pretty good one to study for the GMAT.

@giorgio: what I was trying to show with the numbers above is that it doesn't have to be true that both of the other two countries experienced proportional increases in population greater than Korvo, and that's why we can eliminate the other choice. So, although it could be true that both Guadar and Mitro did, it doesn't have to be true. But it does have to be true that at least one of them did. If both Guadar and Mitro stayed the same or decreased, then Korvo's numerical population increase would have also meant an increase in share.
Kaplan Teacher in Toronto

Master | Next Rank: 500 Posts
Posts: 182
Joined: Sat Apr 11, 2009 4:55 am
Thanked: 11 times

by Giorgio » Fri Jan 15, 2010 1:31 am
Testluv !

Thank you so much , now I understood the question perefectly.

This was quite though !

Legendary Member
Posts: 1404
Joined: Tue May 20, 2008 6:55 pm
Thanked: 18 times
Followed by:2 members

by tanviet » Fri Feb 26, 2010 9:24 pm
we can not have E right because

if total revenue decline, we can not have E while we still have evidence

the question is not clear..

is that right?

GMAT Instructor
Posts: 1302
Joined: Mon Oct 19, 2009 2:13 pm
Location: Toronto
Thanked: 539 times
Followed by:164 members
GMAT Score:800

by Testluv » Sat Feb 27, 2010 1:32 pm
the question is not clear..
No, the question is fine. One way we know this is that it is an official LSAT question.
we can not have E while we still have evidence
actually, we can. But even if we couldn't that wouldn't mean the question is bad; instead, it would mean that choice E is not a proper inference.
Kaplan Teacher in Toronto

Legendary Member
Posts: 1404
Joined: Tue May 20, 2008 6:55 pm
Thanked: 18 times
Followed by:2 members

by tanviet » Sat Feb 27, 2010 6:48 pm
Testluv

if revenue declines by haft and Korva population dose not grew more than of population of one of the 2, we still have Korva percentage of revenue declined

E can not be infered

GMAT Instructor
Posts: 1302
Joined: Mon Oct 19, 2009 2:13 pm
Location: Toronto
Thanked: 539 times
Followed by:164 members
GMAT Score:800

by Testluv » Sat Feb 27, 2010 7:07 pm
We know that each region in the Ditrama federation receives a revenue share proportionally equivalent to that region's contribution to the federation's total population.

We are told that (a) Korva's share in revenue decreased even though (b) Korva's population increased. The only way to reconcile these two facts is that at least one of the other two regions experienced an even greater population increase than did Korva.

If the other two regions' population remained stable (or decreased), Korva's population increase would have also meant an increase in Korva's share of the federation's revenues. Thus, choice E can be properly inferred.
if revenue declines by haft and Korva population dose not grew more than of population of one of the 2, we still have Korva percentage of revenue declined
If the federation's total revenues decline by half, then of course each region would also receive half of what it would have. But this wouldn't impact each region's share in (ie, percentage of) federation revenues.
Kaplan Teacher in Toronto

Master | Next Rank: 500 Posts
Posts: 182
Joined: Mon Apr 20, 2009 7:09 pm
Thanked: 1 times
Followed by:1 members

by akahuja143 » Sat Feb 27, 2010 8:33 pm
thanks for explaination Testluv,I was more inclined towards B at first glance.